LSAT and Law School Admissions Forum

Get expert LSAT preparation and law school admissions advice from PowerScore Test Preparation.

 Administrator
PowerScore Staff
  • PowerScore Staff
  • Posts: 8950
  • Joined: Feb 02, 2011
|
#26208
Complete Question Explanation

Flaw in the Reasoning. The correct answer choice is (A)

The author of this stimulus begins by stating a position of a group of citizens, and then using the “some people say…” rhetorical device, introducing the view of those citizens who oppose turning a railroad grade into a hiking trail. The issue raised by those citizens is that the people who use the trail will probably leave food wrappers and other trash there, littering the area. The author disagrees with the citizens’ premise, claiming that most of the people using the trail will be dedicated hikers, people who care about the environment. Based on this evidence, that the citizens are wrong when they say the trail users will litter the area, the author concludes that the trail should be developed.

This argument displays an error in the use of evidence. Even assuming that the author is correct that most of the people using the trail are dedicated hikers, and even assuming that the trail will not become littered, the author has only weakened the citizens’ argument that the trail should not be constructed. Weakening an argument is not the same as proving its conclusion to be false. However, by concluding that the trail should be developed, the author essentially says that since the citizens’ support is weak, their conclusion is false.

This is a Flaw question. The correct answer choice will describe the author’s misuse of evidence, treating weakening an argument as if it were the same thing as proving the conclusion to be false.

Answer choice (A): This is the correct answer choice, because it describes the author’s assumption that weakening an argument’s support is equivalent to proving that its conclusion is false.

Answer choice (B): This answer choice describes an Error of Composition, in which the characteristic of a member or part of a group is ascribed to the group as a whole, which did not occur in this argument.

Answer choice (C): Here, the answer choice refers to a Circular Argument, in which the evidence offered in support of the conclusion is merely a restatement of the conclusion. In this case, however, the premise and conclusion were distinct.

Answer choice (D): This answer choice indicates an Overgeneralization from a potentially atypical example, but the author’s conclusion was not about an attribute of a majority of users of the trail. The premise discussed an attribute of a majority of the trail’s users, but not the conclusion. For this answer choice to be correct, the conclusion would have needed to say something about the majority of trail users.

Answer choice (E): In this case, the answer choice describes a Source Argument. However, the author did not address the actions or motives of the citizens who oppose the trail, and so this was not a Source Argument.
 Basia W
  • Posts: 108
  • Joined: Jun 19, 2014
|
#16847
Good evening,

I chose D for this question (and after looking on what people on the actual test day scored this seemed to be a very popular answer choice). Is it incorrect from its use of "few users" when the stimulus says most? After I was correcting the exam I looked at the conclusion "but, this objection is groundless"- is this what the correct answer choice is really addressing?

Thank you!

Best,

Basia
 Robert Carroll
PowerScore Staff
  • PowerScore Staff
  • Posts: 1819
  • Joined: Dec 06, 2013
|
#16900
Basia,

There is no attempted inference in the stimulus from "few" to "most." The claim about dedicated hikers was already about "most trail users," so that evidence was already about most. So the author started from most and showed something about most - there's no error there!

The argument is flawed because it has only demonstrated a weakness of one argument against the trail, not proved that the trail should be developed. Even if the argument against is weak, there could be a better argument against. Instead of concluding that the argument is weak, the author concluded that the weak argument's conclusion was false...something not demonstrated. This is why answer choice (A) is correct.

Robert Carroll
 mokkyukkyu
  • Posts: 97
  • Joined: Aug 17, 2016
|
#29480
Are B and D saying similar thing? :(
 Adam Tyson
PowerScore Staff
  • PowerScore Staff
  • Posts: 5376
  • Joined: Apr 14, 2011
|
#29513
Similar, yes, but not identical. B describes an error of composition - the members of a set have a characteristic, so the set must also have that characteristic. That's like saying all the ingredients in my dinner are delicious, so the dinner must also be delicious. That's not necessarily true if the ingredients combine in a way that is not delicious.

D describes an overgeneralization, ascribing a characteristic of some members of a set to the majority of members of the set. That's like saying some of my ingredients are delicious, therefore most of my ingredients are delicious. That's not about the combination (the whole dinner), but about going from "some" to "most".

I think it's fair to say that an error of composition is, in a way, a type of overgeneralization, so the two answers are somewhat similar. They also share one important trait in this case, and that is that neither of them describes the real problem in the stimulus, and are thus both incorrect answer choices.
 ScholesFan
  • Posts: 13
  • Joined: Dec 29, 2018
|
#63440
Hi PowerScore,

I selected D for this question, as well. I'd love a little clarification on a couple of things.

First, part of the reason I didn't choose A is that it's a little vague. I think I would've been more likely to select it if it had read something like this: "bases its conclusion mainly on a sinlge claim that an opposing argument is weak." The way it reads on the test doesn't even sound like it's describing a flaw. Don't we all base disagreements on the belief that the opposition's argument is weak?

Second, concerning D, I guess I assumed that the people in favor of building the trail based their viewpoint on their experience with dedicated hikers. The stimulus doesn't mention this, of course, but surely they've encountered "a few" dedicated hikers before, and that's how they've arrived at their conclusion on this topic. Also, I think the future tense "...will characterize..." made D seem more attractive. It seems to align correctly with the stimulus, which is also based on assumed future behavior.

Any insight you can pass along on overcoming these objections would be greatly appreciated. Thanks in advance. :-D

Thanks,
ScholesFan
 Robert Carroll
PowerScore Staff
  • PowerScore Staff
  • Posts: 1819
  • Joined: Dec 06, 2013
|
#63474
Scholes,

I disagree that we base disagreements on the belief that the opposition's argument is weak. In fact, I think that very often, the situation is reversed. In other words, very often, we think opposing arguments are weak because we already disagree with their conclusions. Regardless of our actual behavior, what's relevant in a Flaw in the Reasoning context is whether our reasoning is good. I want to show why an argument like "Your argument is weak, so I reject its conclusion" is a bad way to argue.

Imagine that someone argues as follows:

"An answer choice on the test that uses the word "only" is usually wrong.
Answer choice (B) uses the word "only".
Therefore, answer choice (B) is wrong."

Not a very good argument. You could correctly describe this argument as "an argument that answer choice (B) is wrong". So, if someone were trying to argue that answer choice (B) is right, THAT person's argument could correctly be described as "an argument that answer choice (B) is right" or as "an argument opposing the argument above."

So...for the advocate of answer choice (B), would this really be a good argument for (B)?

"The quoted argument above, which claims that answer choice (B) is wrong, is fundamentally flawed.
Thus, answer choice (B) is correct."

Absolutely not! What this shows is that pointing out that a particular argument is weak does nothing to prove that that argument's conclusion is wrong. This is the exact situation we have here, and that is why answer choice (A) is correct for this question.

Your explanation of answer choice (D) involves an assumption you're making. That is enough for me to reject your reasoning - this is a Flaw question. We're describing the argument as it was, not changing it or filling in the details we think should have been added. If you have to add information to the stimulus to get answer choice (D) to fit the argument, then that's a proof that the stimulus as given didn't match. That disposes of answer choice (D).

Robert Carroll
 tizwvu34
  • Posts: 20
  • Joined: Aug 15, 2016
|
#64416
I feel like D is an attractive wrong answer because it is describing a flaw that sort of combines parts of both the group of citizens argument and the authors argument. Is the tricky part here deciphering that there are two distinct arguments given in this stimulus, that of the group of citizens and that of the authors, and that the authors argument is simply saying, no your argument won't happen because the people will be regular hikers?
 Brook Miscoski
PowerScore Staff
  • PowerScore Staff
  • Posts: 418
  • Joined: Sep 13, 2018
|
#64430
tizwvu34,

You're right that this one is tricky. I don't believe that (D) is an attractive wrong choice, because D talks about the attributes of a "few users." The stimulus was focused on the attributes of "most" of the users. Therefore, (D) is unsupported.

(B) is very attractive along the line of reasoning that you indicate. In common parlance, "most...have great concern for the environment..." and the conclusion that follows would be heard as a flawed assertion that what most hikers do will be what all hikers do.

However, here the issue is that the stimulus doesn't explicitly assert that "most" hikers will define the group behavior, you have to be wary of (B). The author, as I think you're pointing out, only explicitly pointed out a weakness, he didn't follow through with more. That's why you choose (A).

Another way of explaining (A) is the author only showed that one critique was wrong, not that the plan was good. There could be other reasons not to do the trail, beyond the question of litter. You can't prove your point by attacking your weakest opponent, and that's the flaw that happened here.
 tizwvu34
  • Posts: 20
  • Joined: Aug 15, 2016
|
#64461
Brook,

Thanks, I was mainly going by the scoring %'s in the student center and trying to figure out what people liked about D so much. It's rare to see such a high % of people select the same wrong answer, and have that selection be higher than the correct answer. The % breakdown is:
A) 27%
B) 16%
C) 7%
D) 45%
E) 1 %

Get the most out of your LSAT Prep Plus subscription.

Analyze and track your performance with our Testing and Analytics Package.